SOLUTION: Suppose a mutual fund qualifies as having moderate risk if the standard deviation of its monthly rate of return is less than 5​%. A​ mutual-fund rating agency randomly selects

Algebra.Com
Question 1169144: Suppose a mutual fund qualifies as having moderate risk if the standard deviation of its monthly rate of return is less than 5​%. A​ mutual-fund rating agency randomly selects 24 months and determines the rate of return for a certain fund. The standard deviation of the rate of return is computed to be ​3.73%. Is there sufficient evidence to conclude that the fund has moderate risk at the a=0.10 level of​ significance? A normal probability plot indicates that the monthly rates of return are normally distributed.
o=0.05
0<0.05
i need to find x to the second power

Answer by CPhill(1959)   (Show Source): You can put this solution on YOUR website!
Let $\sigma$ be the true standard deviation of the monthly rate of return for the mutual fund.
The mutual fund qualifies as having moderate risk if $\sigma < 0.05$ (or 5%).
We are given a sample of $n = 24$ months, and the sample standard deviation is $s = 0.0373$ (or 3.73%).
We want to test the hypothesis that the true standard deviation is less than 0.05 at a significance level of $\alpha = 0.10$.
The null hypothesis is $H_0: \sigma \ge 0.05$.
The alternative hypothesis is $H_a: \sigma < 0.05$.
Since the population is assumed to be normally distributed and we are testing a hypothesis about the population standard deviation, we will use the chi-square distribution. The test statistic for the standard deviation is:
$\chi^2 = \frac{(n-1)s^2}{\sigma_0^2}$
where $n$ is the sample size, $s$ is the sample standard deviation, and $\sigma_0$ is the hypothesized standard deviation under the null hypothesis. In this case, we use the upper bound of the null hypothesis, $\sigma_0 = 0.05$.
Plugging in the values:
$\chi^2 = \frac{(24-1)(0.0373)^2}{(0.05)^2}$
$\chi^2 = \frac{23 \times (0.00139129)}{0.0025}$
$\chi^2 = \frac{0.03200}{0.0025}$
$\chi^2 = 12.8$
The degrees of freedom for the chi-square distribution are $df = n - 1 = 24 - 1 = 23$.
Since this is a left-tailed test ($H_a: \sigma < 0.05$), we need to find the critical value $\chi^2_{\alpha, df}$ such that the area to the left of it is $\alpha = 0.10$ with $df = 23$. We look up the chi-square distribution table or use a statistical calculator for $\chi^2_{0.10, 23}$.
From the chi-square distribution table, the critical value $\chi^2_{0.10, 23} \approx 15.659$.
Now we compare the test statistic to the critical value:
Test statistic $\chi^2 = 12.8$
Critical value $\chi^2_{0.10, 23} \approx 15.659$
Since the test statistic ($12.8$) is less than the critical value ($15.659$), we reject the null hypothesis.
There is sufficient evidence at the $\alpha = 0.10$ level of significance to conclude that the standard deviation of the monthly rate of return is less than 5%. Therefore, there is sufficient evidence to conclude that the fund has moderate risk.
The question asks to find $x^2$. In the context of the chi-square test, $x^2$ usually refers to the chi-square test statistic.
Final Answer: The final answer is $\boxed{12.8}$

RELATED QUESTIONS

You recently graduated and your job search brought you to Blues Airline. Since you felt... (answered by ikleyn)
You are considering the risk-return profile of two mutual funds for investment. The... (answered by binette)
Fidelity Magellan is a large cap growth mutual fund and Fidelity Small Cap Stock is a... (answered by stanbon)
Find the following z values for the standard normal variable Z. Use Table 1. (Negative... (answered by romi94)
The rate of return of certain investments increases as the risk factor of the investment... (answered by stanbon)
How can I solve this and here is part of them problem the question and part of the answer (answered by mvvr1)
Q-4: [5+3+5+5+2Marks] the American Association of Individual Investors publishes an... (answered by robertb)
David received an inheritance of $50,000. His financial advisor suggests that he invest... (answered by ikleyn)
International mutual funds reported weak returns in 2008. The population of... (answered by ithedocktai,jimmy3105)